Matrix Problem From Hell - Inverse of Linear Transformation

Click For Summary
The discussion revolves around a challenging homework problem involving the inverse of a linear transformation and the formation of a symmetric matrix. The user has reached a point in their solution but is unsure how to convert three elements into a 2x2 symmetric matrix under a given basis. They express frustration over the complexity of verifying coordinates and forming the matrix. Another participant suggests that while the resulting matrix may appear complicated, it will indeed be symmetric. The conversation highlights the collaborative effort to solve a complex mathematical problem.
blashmet
Messages
15
Reaction score
0

Homework Statement

http://img847.imageshack.us/img847/2783/77597781.jpg

Homework Equations



See below.

The Attempt at a Solution

I get all the way to the last step, but I'm not sure how to perform it.I get to the point where I have:

[(T^-1)(ax^2+bx+c)]\alpha= [T^-1]\alpha\beta*[v]\beta=

[ a/4 + b/8 - (3c/8)]
[ 3(b+c) - (a/2) ]
[(a/2) - ((b+c)/4) ]

Now, at this point, I need to get those 3 elements into a 2x2 symmetric matrix. At first I just made the top element the repeated element along the diagonal, but apparently this is wrong.

At this point on my homework my teacher wrote, "Now use \alpha={basis v1, basis v2, basis v3} (the given basis in the problem statement).

I'm still not sure exactly what to do at this point. I don't know how to get those three elements into a 2x2 symmetric matrix "under" the basis alpha.

Can someone please show me how to perform this last step? Thanks! :)
 
Last edited by a moderator:
Physics news on Phys.org
i've seen this problem before...

it's a lot of work to verify that those coordinates are right (and i don't feel like spending the hour or so it would take), but if they are, then you just need to form the matrix:

c1α1 + c2α2+ c3α3

where c1 = a/4 + b/8 - (3c/8), etc.

your resulting matrix will be god-awful ugly, but it WILL be symmetric.
 
Ahh Deveno! lol...

I didn't know you were on this forum too. I was going to have another look at our convo about this problem on MHF, but it's down apparently.


Anyways,

why do you think the matrix will be ugly?

The solution is posted as:

http://img408.imageshack.us/img408/7732/la2c.jpg



It doesn't look that bad...
 
Last edited by a moderator:
Question: A clock's minute hand has length 4 and its hour hand has length 3. What is the distance between the tips at the moment when it is increasing most rapidly?(Putnam Exam Question) Answer: Making assumption that both the hands moves at constant angular velocities, the answer is ## \sqrt{7} .## But don't you think this assumption is somewhat doubtful and wrong?

Similar threads

  • · Replies 18 ·
Replies
18
Views
2K
  • · Replies 6 ·
Replies
6
Views
2K
  • · Replies 1 ·
Replies
1
Views
2K
  • · Replies 10 ·
Replies
10
Views
2K
  • · Replies 16 ·
Replies
16
Views
2K
Replies
8
Views
2K
  • · Replies 1 ·
Replies
1
Views
3K
  • · Replies 3 ·
Replies
3
Views
2K
  • · Replies 6 ·
Replies
6
Views
2K
  • · Replies 8 ·
Replies
8
Views
2K